ChaseDream

标题: OG-119 借此题与LAWER讨论一下WEAKEN [打印本页]

作者: 我爱欧洲    时间: 2005-5-8 11:23
标题: OG-119 借此题与LAWER讨论一下WEAKEN

According to the Tristate Transportation Authority, making certain improvements to the main commuter rail line
would increase ridership dramatically. The authority plans to finance these improvements over the course of five
years by raising automobile tolls on the two high-way bridges along the route the rail line serves. Although the
proposed improvements are indeed needed, the authority’s plan for securing the necessary funds should be
rejected because it would unfairly force drivers to absorb the entire cost of something from which they receive no
benefit.
119. Which of the following, if true, would cast the most doubt on the effectiveness of the authority’s plan to
finance the proposed improvements by increasing bridge tolls?
(A) Before the authority increases tolls on any of the area bridges, it is required by law to hold public hearings at
which objections to the proposed increase can be raised.
(B) Whenever bridge tolls are increased, the authority must pay a private contractor to adjust the automated
toll-collecting machines.
(C) Between the time a proposed toll increase is announced and the time the increase is actually put into effect,
many commuters buy more tokens than usual to postpone the effects of the increase.
(D) When tolls were last increased on the two bridges in question, almost 20 percent of the regular commuter
traffic switched to a slightly longer alternative route that has since been improved.
(E) The chairman of the authority is a member of the Tristate Automobile Club that has registered strong
opposition to the proposed toll increase.



答案是D没问题。A和E据OG解释Choices A and E suggest that the plan might face opposition but not that it will be defeated not that the
anticipated revenue will not be generated. Therefore neither A nor E is correct.LAWER在OG67中有这么一句话“对WEAKEN题,问自己,是否该选项迫使作者重新考虑他的观点或必须作出反应。否则他的观点就要受怀疑。”当时我非常同意,我也觉得,WEAKEN选项的目的是在成立的情况下,可以或可能造成WEAKEN的结果,并不一定造成WEAKEN的结果!但A和E两个选项按OG的解释可能造成反对该计划的效果,确实“迫使作者重新考虑他的观点或必须作出反应。否则他的观点就要受怀疑”,按LAWER的理论不也就造成削弱效果了吗?


作者: luxing    时间: 2005-5-8 15:51

I give below some of my instinct feelings on this for your reference:

“WEAKEN选项的目的是在成立的情况下,可以或可能造成WEAKEN的结果,并不一定造成WEAKEN的结果”, 我觉得WEAKEN选项的目的是在成立的情况下,一定造成WEAKEN的结果!!

任何事都可能有反对的声音,但是关键是看是否动摇了立论的基础:

A选项是举行公共听证会 -- 可以有反对,但是也可能没有反对。

E选项中Tristate Automobile Club 的成员,未必就会有同Tristate Automobile Club 一样的立场。两个选项都有不确定性。

但是D选项否定了文章的立论基础- raising funds to finance the improvements。


作者: 我爱欧洲    时间: 2005-5-8 23:15
谢谢兄弟解释。但LAWER的话到底怎样理解我还是不清楚,LAWER兄能否解释一下自己的话呢?
作者: lawyer_1    时间: 2005-5-9 12:21
A,E削弱的是该计划能否实行,而问题要你削弱的是该计划实行的效果(前提是该计划已实行)。所以无关。
作者: lawyer_1    时间: 2005-5-9 12:29
既然无关,作者根本不必理会AE,即使AE的情况是真的,他的观点也不必受到怀疑。请注意作者的观点是:用增加过桥费的办法可以为改进主干线筹集到资金。是在讲计划的效果如何。而AE讲的该计划是否能实行得了。
作者: 我爱欧洲    时间: 2005-5-9 12:50

很有道理,NN的话总是一针见血。这题我明白了!

但还是想请问LAWER,我在1楼对你的话的理解是否正确?

还有“OG总章”这种东西在哪能找到?WEAKEN,ASSUMPTION之类官方解释能在哪找到?


作者: kemu8018    时间: 2006-9-2 03:09
up up..
作者: cccccc0    时间: 2006-11-11 14:05
up
作者: KATIEUS    时间: 2006-11-29 04:48





欢迎光临 ChaseDream (https://forum.chasedream.com/) Powered by Discuz! X3.3